Solutions détaillées de neuf exercices sur la dérivation des fonctions (fiche 02)
Cliquer ici pour accéder aux énoncés

icone-math-OS-Exos
exercice 1 facile

L’application f est définie et continue sur \mathbb{R}, dérivable au moins sur \mathbb{R}^{\star}.
Calculons son taux d’accroissement en 0. Pour tout x\neq0 :

    \[T_{f,0}\left(x\right)=\frac{f\left(x\right)-f\left(0\right)}{x}=\frac{1}{\left|x\right|+1}\]

donc :

    \[\lim_{x\rightarrow0}T_{f,0}\left(x\right)=1\]

Ceci montre que f est dérivable en 0 et que f'\left(0\right)=1.

A titre indicatif, voici l’allure du graphe de f, qui est symétrique par rapport à l’origine (puisque f est impaire) et admet la première bissectrice pour tangente à l’origine. En outre, on peut signaler les deux asymptotes horizontales, d’équations y=-1 et y=1.

Remarque

On sait que la composée de deux applications dérivables est dérivable.

L’intérêt de cet exercice est de montrer que la dérivabilité d’une composée g\circ f n’implique pas celle de f et de g. Sauriez-vous trouver un exemple de couple \left(f,g\right) d’applications non dérivables tel que g\circ f soit dérivable ?

Pour dévoiler une solution (parmi tant d’autres) … cliquer ici

Considérons la fonction indicatrice de \mathbb{Q} :

    \[f:\mathbb{R}\rightarrow\mathbb{R},x\mapsto\left\{\begin{array}{cc}1 & \text{si }x\in\mathbb{Q}\\0 & \text{sinon}\end{array}\right.\]

Il est bien connu que f est discontinue en tout point (et qu’elle n’est donc dérivable nulle part). Pourtant f\circ f est l’application constante x\mapsto1 qui est dérivable en tout point.

exercice 2 facile

L’application \left]-1,1\right[\rightarrow\mathbb{R},\thinspace x\mapsto{\displaystyle \frac{x}{\sqrt{1-x^{2}}}} est dérivable et donc f aussi (composée d’applications dérivables). Pour tout x\in\left]-1,1\right[ :

    \[f'\left(x\right)=\frac{\sqrt{1-x^{2}}-x\:{\displaystyle \frac{-2x}{2\sqrt{1-x^{2}}}}}{1-x^{2}}\:\arctan'\left(\frac{x}{\sqrt{1-x^{2}}}\right)\]

c’est-à-dire :

    \[f'\left(x\right)=\frac{1}{\left(1-x^{2}\right)^{3/2}}\:\frac{1}{1+{\displaystyle \frac{x^{2}}{1-x^{2}}}}\]

soit finalement :

    \[f'\left(x\right)=\frac{1}{\sqrt{1-x^{2}}}\]

Il s’ensuit l’existence d’un réel A tel que :

    \[\forall x\in\left]-1,1\right[,\thinspace f\left(x\right)=\arcsin\left(x\right)+A\]

et l’on voit, en évaluant en x=0, que A=0. En conclusion :

    \[\boxed{\forall x\in\left]-1,1\right[,\thinspace\arctan\left(\frac{x}{\sqrt{1-x^{2}}}\right)=\arcsin\left(x\right)}\]

Remarque 1

Ce calcul intervient dans la preuve du lemme donnée à l’annexe 2 de cet article.

Remarque 2

Il existe une autre façon de traiter cet exercice, sans passer par un calcul de dérivée. Etant donné x\in\left]-1,1\right[, on considère l’unique \alpha\in\left]-\frac\pi2,\frac\pi2\right[ tel que x=\sin(\alpha). On voit alors que :

    \[\arctan\left(\frac{x}{\sqrt{1-x^2}}\right)=\arctan\left(\tan(\alpha)\right)=\alpha=\arcsin(x)\]

exercice 3 facile

Supposons f' impaire et montrons que f est paire. Pour cela, posons pour tout x\in\mathbb{R} :

    \[g\left(x\right)=f\left(x\right)-f\left(-x\right)\]

Comme f est dérivable, alors g l’est aussi et, pour tout x\in\mathbb{R} :

    \[g'\left(x\right)=f'\left(x\right)+f'\left(-x\right)=0\]

donc g est constante. Mais g\left(0\right)=0 et donc g est identiquement nulle. Autrement dit f est paire.

A présent, supposons f' paire. On pourrait penser que cette hypothèse va entraîner l’imparité de f, mais il n’en est rien (contre-exemple avec f:x\mapsto x+1). Cependant, on peut adapter le calcul précédent et considérer l’application h définie par :

    \[\forall x\in\mathbb{R},\thinspace h\left(x\right)=f\left(x\right)+f\left(-x\right)\]

On voit que h' est nulle, donc h est constante et il existe B\in\mathbb{R} tel que :

    \[\forall x\in\mathbb{R},\thinspace f\left(x\right)+f\left(-x\right)=B\]

Ceci exprime, pour le graphe de f, l’existence d’un centre de symétrie, à savoir le point \left(0,{\displaystyle \frac{B}{2}}\right).

Remarque

Toutefois, si f' est paire et si de plus f\left(0\right)=0, alors f est impaire.

Supposons f'\left(0\right)=0. Pour tout n\in\mathbb{N}^{\star} et pour tout x\in\mathbb{R} :

    \[\left(f^{n}\right)'\left(x\right)=\left(f^{n-1}\circ f\right)'\left(x\right)=\left(f^{n-1}\right)'\left(f\left(x\right)\right)\:\times\:f'\left(x\right)\]

En particulier :

    \[\boxed{\left(f^{n}\right)'\left(0\right)=0}\]

Supposons maintenant f\left(0\right)=0 et f'\left(0\right)=1. Alors :

    \[\left(f^{n}\right)'\left(0\right)=\left(f^{n-1}\right)'\left(0\right)\]

d’où (par une récurrence immédiate) \left(f^{n}\right)'\left(0\right)=f'\left(0\right), soit :

    \[\boxed{\left(f^{n}\right)'\left(0\right)=1}\]

D’après la formule de Leibniz (voir cet article). Pour tout x\in\mathbb{R} et pour tout n\in\mathbb{N} :

    \[f^{\left(n\right)}\left(x\right)=\sum_{k=0}^{n}\binom{n}{k}\exp^{\left(n-k\right)}\left(x\right)\sin^{\left(k\right)}\left(x\right)\]

c’est-à-dire :

    \[f^{\left(n\right)}\left(x\right)=e^{x}\:\sum_{k=0}^{n}\binom{n}{k}\sin\left(x+\frac{k\pi}{2}\right)\]

et donc, en particulier :

    \[f^{\left(n\right)}\left(0\right)=\sum_{k=0}^{n}\binom{n}{k}\sin\left(\frac{k\pi}{2}\right)\]

Les termes d’indices pairs sont tous nuls et il reste donc :

    \[f^{\left(n\right)}\left(0\right)=\sum_{q=0}^{\left\lfloor \frac{n-1}{2}\right\rfloor }\binom{n}{2q+1}\sin\left(q\pi+\frac{\pi}{2}\right)=\sum_{q=0}^{\left\lfloor \frac{n-1}{2}\right\rfloor }\left(-1\right)^{q}\binom{n}{2q+1}\]

On pourrait en rester là, mais il est tentant d’essayer de trouver une formule close pour cette dernière somme.

Voici l’astuce, qui fait un petit crochet par le champ complexe … On observe que :

    \[\sum_{k=0}^{n}\binom{n}{k}i^{k}=\left(1+i\right)^{n}=2^{n/2}\thinspace e^{in\pi/4}\]

donc, en passant aux parties imaginaires :

    \[\sum_{q=0}^{\left\lfloor \frac{n-1}{2}\right\rfloor }\binom{n}{2q+1}\left(-1\right)^{q}=2^{n/2}\sin\left(\frac{n\pi}{4}\right)\]

Finalement :

    \[\boxed{\forall n\in\mathbb{N},\thinspace f^{\left(n\right)}\left(0\right)=2^{n/2}\sin\left(\frac{n\pi}{4}\right)}\]

Cela dit, puisqu’on s’est autorisé à passer par les complexes, autant y aller carrément ! On peut d’emblée observer que :

    \[\forall x\in\mathbb{R},\thinspace f\left(x\right)=\text{Im}\left(e^{\left(1+i\right)x}\right)\]

Or, on sait que pour une fonction dérivable à valeurs complexes, la dérivée de la partie imaginaire est égale à la partie imaginaire de la dérivée … et donc :

    \[f^{\left(n\right)}\left(x\right)=\text{Im}\left(\left(1+i\right)^{n}\thinspace e^{\left(1+i\right)x}\right)\]

d’où en particulier :

    \[f^{\left(n\right)}\left(0\right)=\text{Im}\left(\left(1+i\right)^{n}\right)\]

et l’on retombe ainsi plus rapidement (et sans avoir besoin de la formule de Leibniz) sur le résultat encadré plus haut.

Lorsque x est proche de 0, la variable d’intégration parcourt \left[x,2x\right] donc se promène aussi au voisinage de 0 et donc e^{t} est voisin de 1. On peut donc conjecturer que f\left(x\right) se comporte, lorsque x tend vers 0, comme :

    \[\int_{x}^{2x}\frac{1}{t}\thinspace dt=\ln\left(2\right)\]

Pour le prouver, on regarde la différence :

    \[f\left(x\right)-\ln\left(2\right)=\int_{x}^{2x}\frac{e^{t}-1}{t}\thinspace dt\]

Cette quantité est positive (intégrale d’une fonction positive) et comme l’application {\displaystyle t\mapsto\frac{e^{t}-1}{t}} admet une limite finie (égale à 1) en 0, elle est bornée sur \left]0,1\right]. Il existe M>0 tel que :

    \[\forall x\in\left]0,\frac{1}{2}\right],\thinspace0\leqslant f\left(x\right)-\ln\left(2\right)\leqslant Mx\]

Cet encadrement prouve que :

    \[\boxed{\lim_{x\rightarrow0^{+}}f\left(x\right)=\ln\left(2\right)}\]

On peut donc prolonger f par continuité en 0 en posant :

    \[\tilde{f}:\left[0,+\infty\right[\rightarrow\mathbb{R},\thinspace x\mapsto\left\{\begin{array}{cc}\ln\left(2\right) & \text{si }x=0\f\left(x\right) & \text{si }x>0\end{array}\right.\]

On va maintenant s’intéresser à la dérivabilité de \tilde{f} en 0. Posons pour tout x>0 :

    \[G\left(x\right)=\int_{1}^{x}\frac{e^{t}}{t}\thinspace dt\]

G est la primitive de \left]0,+\infty\right[\rightarrow\mathbb{R},\thinspace{\displaystyle x\mapsto\frac{e^{x}}{x}} qui s’annule en 1. D’après la relation de Chasles, pour tout x>0 :

    \[f\left(x\right)=G\left(2x\right)-G\left(x\right)\]

donc f est dérivable sur \left]0,+\infty\right[ et pour tout x>0 :

    \[f'\left(x\right)=2\thinspace G'\left(2x\right)-G'\left(x\right)=\frac{e^{2x}-e^{x}}{x}\]

En utilisant un développement limité au 1er ordre pour l’exponentielle, on voit que :

    \[\lim_{x\rightarrow0^{+}}f'\left(x\right)=1\]

On en déduit, grâce au théorème de la limite de la dérivée, que \tilde{f} est dérivable en 0 et que :

    \[\boxed{\tilde{f}'\left(0\right)=1}\]

Pour x>0, l’intégrale

    \[\int_{0}^{+\infty}\frac{\sin\left(t\right)}{x+t}\thinspace dt\]

est impropre pour la borne +\infty. On voit avec une intégration par parties que, si 0<x<T alors :

    \begin{eqnarray*}\int_{0}^{T}\frac{\sin\left(t\right)}{x+t}\thinspace dt & = & \left[\frac{-\cos\left(t\right)}{x+t}\right]_{t=0}^{T}-\int_{0}^{T}\frac{\cos\left(t\right)}{\left(x+t\right)^{2}}\thinspace dt\\& = & \frac{1}{x}-\frac{\cos\left(T\right)}{x+T}-\int_{0}^{T}\frac{\cos\left(t\right)}{\left(x+t\right)^{2}}\thinspace dt \end{eqnarray*}

Comme l’intégrale {\displaystyle \int_{0}^{+\infty}\frac{\cos\left(t\right)}{\left(x+t\right)^{2}}\thinspace dt} est absolument convergente, elle est convergente et donc l’intégrale partielle {\displaystyle \int_{0}^{T}\frac{\sin\left(t\right)}{x+t}\thinspace dt} admet, lorsque T tend vers +\infty, une limite finie à savoir :

    \[ \lim_{T\rightarrow+\infty}\int_{0}^{T}\frac{\sin\left(t\right)}{x+t}\thinspace dt=\frac{1}{x}-\int_{0}^{+\infty}\frac{\cos\left(t\right)}{\left(x+t\right)^{2}}\thinspace dt \]

Bref, f est bien définie. Ensuite, le changement de variable s=x+t donne :

    \[ f\left(x\right)=\int_{x}^{+\infty}\frac{\sin\left(s-x\right)}{s}\thinspace ds \]

et donc, d’après la formule d’addition pour le sinus :

    \[ f\left(x\right)=\cos\left(x\right)\int_{x}^{+\infty}\frac{\sin\left(s\right)}{s}\thinspace ds-\sin\left(x\right)\int_{x}^{+\infty}\frac{\cos\left(s\right)}{s}\thinspace ds \]

Pour alléger un peu l’écriture, notons :

    \[ C\left(x\right)=\int_{x}^{+\infty}\frac{\cos\left(s\right)}{s}\thinspace ds\qquad\text{et}\qquad S\left(x\right)=\int_{x}^{+\infty}\frac{\sin\left(s\right)}{s}\thinspace ds \]

de sorte que :

    \[ f\left(x\right)=\cos\left(x\right)S\left(x\right)-\sin\left(x\right)C\left(x\right) \]

A partir de là, il est facile de dériver :

    \[ f'\left(x\right)=-\sin\left(x\right)S\left(x\right)-\cos\left(x\right)C\left(x\right)+\cos\left(x\right)S'\left(x\right)-\sin\left(x\right)C'\left(x\right) \]

Or :

    \[ C'\left(x\right)=-\frac{\cos\left(x\right)}{x}\qquad\text{et}\qquad S'\left(x\right)=-\frac{\sin\left(x\right)}{x} \]

et donc :

    \[ f'\left(x\right)=-\sin\left(x\right)S\left(x\right)-\cos\left(x\right)C\left(x\right) \]

On dérive encore un coup :

    \begin{eqnarray*} f''\left(x\right) & = & -\cos\left(x\right)S\left(x\right)+\sin\left(x\right)C\left(x\right)-\sin\left(x\right)S'\left(x\right)-\cos\left(x\right)C'\left(x\right)\\ & = & -f\left(x\right)+\frac{\sin^{2}\left(x\right)+\cos^{2}\left(x\right)}{x}\\ & = & -f\left(x\right)+\frac{1}{x} \end{eqnarray*}

En définitive, on a prouvé que :

    \[ \boxed{\forall x>0,\thinspace f''\left(x\right)+f\left(x\right)=\frac{1}{x}}\]

Comme \deg\left(P\right) est impair, alors P possède (au moins) une racine réelle a. L’hypothèse impose :

    \[\forall n\in\mathbb{N},\thinspace f^{\left(n\right)}\left(a\right)=0\]

Pour tout x\in\mathbb{R} et pour tout n\in\mathbb{N}, la formule de Taylor avec reste intégral donne :

    \[f\left(x\right)=\int_{a}^{x}\frac{\left(x-t\right)^{n}}{n!}f^{\left(n+1\right)}\left(t\right)\thinspace dt\]

et donc :

(\star)   \[\left|f\left(x\right)\right|\leqslant\frac{\left|x-a\right|^{n+1}}{n!}\sup\left\{\left|P\left(t\right)\right|;\thinspace t\in\left[a,x\right]\right\}\]

Or, on sait que :

    \[\forall\lambda\in\mathbb{R},\thinspace\lim_{n\rightarrow\infty}\frac{\lambda^{n}}{n!}=0\]

En passant à la limite (pour x fixé et en faisant tendre n vers +\infty) dans \left(\star\right), on conclut que f est l’application nulle.

exercice 9 difficile

On peut traiter cette question en combinant deux outils : d’une part, le lemme de Rolle et, d’autre part, le petit artifice technique suivant …

Si l’on pose, pour tout t\in\mathbb{R} :

    \[h\left(t\right)=f\left(t\right)\thinspace e^{\alpha t}\]

alors :

    \[h'\left(t\right)=\left(f'\left(t\right)+\alpha\thinspace f\left(t\right)\right)\thinspace e^{\alpha t}=g\left(t\right)\thinspace e^{\alpha t}\]

ce qui montre que les zéros de g sont ceux de h'.

Supposons n\geqslant2. Par hypothèse, il existe des réels t_{1}<\cdots<t_{n} tels que, pour tout i\in\left\llbracket 1,n\right\rrbracket , f\left(t_{i}\right)=0 et donc h\left(t_{i}\right)=0. En appliquant le lemme de Rolle à h sur chacun des segments \left[t_{i},t_{i+1}\right], on voit qu’il existe des réels c_{1}<\cdots<c_{n-1} tels que :

    \[\forall i\in\left\llbracket 1,n-1\right\rrbracket ,\thinspace h'\left(c_{i}\right)=0\]

et donc :

    \[\forall i\in\left\llbracket 1,n-1\right\rrbracket ,\thinspace g\left(c_{i}\right)=0\]

Il reste à montrer l’existence d'(au moins) un n-ème zéro pour g. Ceci va découler du résultat suivant :

Lemme de Rolle étendu aux intervalles non bornés

Soit \varphi:\left[a,+\infty\right[\rightarrow\mathbb{R} une application continue. On suppose que \varphi est dérivable sur \left]a,+\infty\right[ et que :

    \[\lim_{t\rightarrow+\infty}\varphi\left(t\right)=\varphi\left(a\right)\]

Alors il existe c\in\left]a,+\infty\right[ vérifiant \varphi'\left(c\right)=0.

Si \alpha<0 alors vu que f est bornée, on voit que h est bornée sur \left[t_{n},+\infty\right[ et donc, en appliquant ce lemme à h sur cet intervalle, on met en évidence un réel c_n tel que c_n>t_n et h'\left(c_n\right)=0, d’où g\left(c_n\right)=0.

De même, si \alpha>0, on procède de même, mais sur l’intervalle \left]-\infty,t_{1}\right], ce qui donne l’existence d’un réel c_0<c_1 tel que g(c_0)=0.

Bref, dans tous les cas, on constate que g s’annule (au moins) n fois.

Si un point n’est pas clair ou vous paraît insuffisamment détaillé, n’hésitez pas à poster un commentaire ou à me joindre via le formulaire de contact.

Partager cet article